Wann gibt es genug Kasimire?

Ich weiß, dass ein Casimir für eine Lie-Algebra g ist ein zentrales Element der universellen Hüllalgebra. Zum Beispiel im s Ö ( 3 ) die Generatoren sind die Drehimpulsoperatoren J 1 , J 2 , J 3 und ein quadratischer Kasimir ist J 2 .

Betrachten Sie eine irreduzible Darstellung d von g an v . Durch Schurs Lemma irgendein Casimir C wird auf eine Matrix mit einem einzigen Eigenwert abgebildet C d . Wenn d und e sind dann äquivalente Darstellungen C d = C e da der Basiswechsel die Eigenwerte erhält.

Unter welchen Umständen gibt es genügend Kasimire, dass wir eine Umkehrung dieser Aussage erhalten? Mit anderen Worten, wann können wir genug Casimirs finden { C d , D d , F d } irreduzible Darstellungen eindeutig kennzeichnet? Ich kann nicht ausrechnen, wie viele wir im Allgemeinen brauchen würden, oder wirklich, wie man das beweist!

Vielen Dank im Voraus.

Dieselbe Frage bei Math.SE: math.stackexchange.com/q/288816/11127

Antworten (2)

Wir diskutieren hier nur den Fall endlichdimensionaler irreduzibler Darstellungen (irreps) einer komplexen halbeinfachen Lie-Algebra L .

Daran erinnern, dass das Set Z von Casimir-Invarianten ist das Zentrum Z ( U ( L ) ) der universellen Hüllalgebra U ( L ) , vgl. zB dieser Phys.SE Beitrag.

Die Frage von OP wird ohne Beweis auf p beantwortet. 253 in Art.-Nr. 1:

Satz 2. Für jede halbeinfache Lie-Algebra L von Rang r , gibt es eine Reihe von r invariantes Polynom des Generators t a , deren Eigenwerte die endlichdimensionalen irreduziblen Darstellungen charakterisieren.

Ref. 2 (das eines der wichtigsten Bücher über Lie-Algebren ist, zumindest wenn man sich für die Beweise interessiert) macht sich nicht die Mühe, Theorem 2 explizit zu erwähnen. Es ist jedoch möglich, eine Reihe grundlegenderer Ergebnisse (und ihrer Beweise) aus Lit. aneinander zu reihen. 2, um das gewünschte Ergebnis zu erhalten. Wir skizzieren die Beweisstrategie unten.

Erinnern Sie sich außerdem daran, dass ein Root-System zugeordnet ist Φ zur Lie-Algebra L , und stellen wir uns vor, dass wir eine Basis ausgewählt haben Δ zum Φ . Die Bestellung | W | der Weyl-Gruppe W ist gleich den Auswahlmöglichkeiten von (ungeordneten) Basen und gleich den Auswahlmöglichkeiten von (fundamentalen) Weyl-Kammern .

Es ist in den Kapiteln 20-21 von Ref. bewiesen. 2. dass ein endlichdimensionaler Irrep einen eindeutig höchsten Gewichtsvektor (eindeutig bis zur Normalisierung) mit einem dominanten integralen Gewicht hat λ . Solche irrep werden wir fortan bezeichnen v ( λ ) . (Ref. 2. definiert auch einen Begriff eines höchsten Gewichts irrep v ( λ ) Wenn λ ist integral, aber nicht dominant. Solche Irreps sind notwendigerweise unendlichdimensional, also werden wir sie ignorieren.) Daraus folgt

Zwei irreps v ( λ ) und v ( μ ) sind äquivalent (dh isomorph), wenn ihre höchsten Gewichte gleich sind λ = μ .

Als Folge des Satzes von Harish-Chandra ist die Menge Z von Casimirs nimmt denselben Wert auf zwei Irreps mit dem höchsten Gewicht an v ( λ ) und v ( μ ) iff λ + δ und μ + δ gehören zur gleichen Weyl-Umlaufbahn,

σ ( λ + δ )   =   μ + δ , σ W .

Hier δ ist die Hälfte der Summe der positiven Wurzeln. Wenn jedoch beide integrale Gewichte λ und μ sind dann dominant λ + δ und μ + δ müssen beide (dem Inneren) der fundamentalen Weyl-Kammer angehören, damit die Weyl-Reflexion σ = 1 muss das Identitätselement sein. Abschließend verstehen wir das

Der Satz Z von Casimirs nimmt denselben Wert auf zwei endlichdimensionalen Irreps an v ( λ ) und v ( μ ) wenn ihre höchsten Gewichte gleich sind λ = μ .

Das Theorem von Harish-Chandra wird in Kapitel 23 von Ref. bewiesen. 2. Siehe auch diesen und diesen verwandten Math.SE-Beitrag.

Beispiel: Betrachten Sie die Lie-Algebra L = s l ( 3 , C ) . Die Weyl-Gruppe ist S 3 . Die Lie-Algebra L hat zwei unabhängige Casimir-Invarianten C 2 und C 3 ,

C n   :=   s t r ( a d t a 1 a d t a n ) t a 1 t a n , n     { 2 , 3 } .

Betrachten Sie die dreidimensionale fundamentale Darstellung F und die duale/kontrarediente Darstellung F ¯ von L , die nicht äquivalente Irreps sind. Sie haben höchste Gewichte λ = ( 1 , 0 ) und μ = ( 0 , 1 ) , beziehungsweise. Im Einzelnen ggf t a , a = 1 , , 8 sind Generatoren für L = s l ( 3 , C ) , dann (hattip: Peter Kravchuk)

F ¯ ( t a )   =   F ( t a ) t ,

damit die Kasimire C 2 (und C 3 ) nehmen den gleichen (entgegengesetzten) Wert an F und F ¯

t r F ¯ F ¯ ( C n )   =   ( 1 ) n t r F F ( C n ) , n     { 2 , 3 } .

Man kann beweisen, dass die Werte ungleich Null sind, so dass die Casimirs C 2 und C 3 unterscheiden zwischen den beiden nicht äquivalenten irreps F und F ¯ , wie sie sollten.

Verweise:

  1. AO Barut und R. Raczka, Theory of Group Representations and Applications, 2. Aufl., 1980.

  2. JE Humphreys, Einführung in die Lie-Algebren und Darstellungstheorie, (1980).

Danke - das ist wunderbar. Ich glaube, ich habe jetzt eine viel bessere Intuition dafür!
@Qmechanic, haben Sie einen Hinweis darauf, dass C 3 ist das gleiche in F und F ¯ ? Es ist für mich einfach nicht offensichtlich, da wir das Dual durch Mapping erhalten F ¯ ( g ) = F ( g ) T , und IIRC C 3 ist in Generatoren sowohl kubisch als auch diagonal, also F ¯ ( C 3 ) = ( 1 ) 3 F ( C 3 ) T = F ( C 3 ) . Oder ist es in beiden Fällen nur Null?
@Qmechanic, ich habe einige Formeln für die Werte von gefunden C 2 und C 3 in Darstellung mit Gewicht ( m 1 , m 2 ) : c 2 = 2 3 ( m 1 2 + m 2 2 + m 1 m 2 + 3 m 1 + 3 m 2 ) und c 3 = 1 9 ( m 1 m 2 ) ( 2 m 1 + m 2 + 3 ) ( 2 m 2 + m 1 + 3 ) die dazu führen F ( C 3 ) = 20 9 und F ¯ ( C 3 ) = 20 9 . Es stammt von Lyahovsky-Bolochov, aber ich bezweifle, dass es eine englische Übersetzung gibt. Meine Behauptung in der gelöschten Antwort stammt ebenfalls aus diesem Buch (nun, ich war nicht genau, die Aussage lautet, dass Casimirs für einfache Algebren zwei beliebige nicht äquivalente Wiederholungen unterscheiden kann). Ich konnte ihre Referenz jedoch noch nicht finden.
@ Peter Kravchuk. Vielen Dank. Ich habe versehentlich zuerst die drei Weyl-Spiegelachsen gezeichnet L = s l ( 3 , C ) falsch (tilted by 30 ), was mich irgendwie dazu verleitet, eine Reihe falscher Schlussfolgerungen zu glauben.
@Qmechanic, ja, ich habe den gleichen Fehler gemacht, aber irgendwann bemerkt.)
@Qmechanic, also gilt der 'andere Fall' des Satzes von Harish-Chandra tatsächlich nur für unendlich dimensionale Module? Ebenfalls, s t r ist die symmetrisierte Spur?
@ Peter Kravchuk: Ja und ja.
@Qmechanic, was passiert also, wenn die Algebra nicht halb einfach ist? Sagen wir eine Poincaré-Algebra - wie sieht man dann, dass es genau 2 Casimir-Generatoren gibt und nicht mehr und dass sie die Irreps eindeutig kennzeichnen?

Es kann bewiesen werden (Theorem von Racah), dass die Anzahl der Casimir-Operatoren gleich dem Rang der Algebra (Anzahl gleichzeitig konmutierender Generatoren) ist. Dies gilt zumindest für halbeinfache Algebren.